LSAT and Law School Admissions Forum

Get expert LSAT preparation and law school admissions advice from PowerScore Test Preparation.

User avatar
 Dave Killoran
PowerScore Staff
  • PowerScore Staff
  • Posts: 5852
  • Joined: Mar 25, 2011
|
#87695
Complete Question Explanation
(The complete setup for this game can be found here: lsat/viewtopic.php?t=1469)

The correct answer choice is (C)

This List question is easily attacked by applying the rules of the game.

Answer choice (A) can be eliminated because from the first rule H and S cannot both be taken.

Answer choice (B) can be eliminated because from the first rule H and M cannot both be taken.

Answer choice (D) can be eliminated because from the third rule W and P cannot both be taken.

Answer choice (E) can be eliminated because from the second rule M and T cannot both be taken.

Thus, answer choice (C) is proven correct by process of elimination.

Get the most out of your LSAT Prep Plus subscription.

Analyze and track your performance with our Testing and Analytics Package.